YOMEDIA
NONE
  • Câu hỏi:

    Cho hàm số bậc 4 \(y=f\left( x \right)=a{{x}^{4}}+b{{x}^{3}}+c{{x}^{2}}+dx+e\) thỏa mãn \(f\left( 0 \right)=3f\left( 2 \right)=-3\) và có đồ thị hàm số \(y={f}'\left( x \right)\) như hình:

    Có bao nhiêu giá trị nguyên của tham số \(m\) thuộc khoảng \(\left( -20;20 \right)\) để hàm số \(g\left( x \right)=f\left[ 4f\left( x \right)-f''\left( x \right)+m \right]\) đồng biến trên khoảng \(\left( 0;1 \right)?\)

    • A. \(30.\)          
    • B. \(29.\)                   
    • C. \(0.\)              
    • D. \(10.\)

    Lời giải tham khảo:

    Đáp án đúng: A

    Xét  \(y=f\left( x \right)=a{{x}^{4}}+b{{x}^{3}}+c{{x}^{2}}+dx+e\Rightarrow f'\left( x \right)=4a{{x}^{3}}+3b{{x}^{2}}+2cx+d\).

    Từ đồ thị \(y={f}'\left( x \right)\) ta có \({f}'(x)=4ax\left( {{x}^{2}}-1 \right)=4a{{x}^{3}}-4ax.\)

    Vậy ta có hệ phương trình

    \(\begin{array}{l} \left\{ \begin{array}{l} b = 0\\ 2c = - 4\\ d = 0 \end{array} \right.a\\ \Leftrightarrow \left\{ \begin{array}{l} b = 0\\ c = - 2a\\ d = 0 \end{array} \right. \end{array}\)

     \(\Rightarrow f\left( x \right)=a{{x}^{4}}-2a{{x}^{2}}+e\).

    Ta lại có

    \(f\left( 0 \right)=3f\left( 2 \right)=-3\\\Rightarrow \left\{ \begin{align} & a=\frac{1}{4} \\ & e=-3 \\ \end{align} \right.\)

    Vậy \(f\left( x \right)=\frac{1}{4}{{x}^{4}}-\frac{1}{2}{{x}^{2}}-3.\)

    Ta có \({f}'(x)={{x}^{3}}-x\)\(\Rightarrow f''\left( x \right)=3{{x}^{2}}-1\)\(\Rightarrow f'''\left( x \right)=6x\)

    Xét hàm số \(g\left( x \right)=f\left( 2f\left( x \right)-f''\left( x \right)+m \right)\) trên đoạn \(\left[ 0;1 \right]\)

    Ta có \(g'\left( x \right)=\left[ 4f'\left( x \right)-f'''\left( x \right) \right]f'\left[ 4f\left( x \right)-f''\left( x \right)+m \right]\)

    Hàm số \(g\left( x \right)\) đồng biến trên khoảng \(\left( 0;1 \right)\) \(\Leftrightarrow g'\left( x \right)\ge 0,\forall x\in \left( 0;1 \right).\)

    Mà \(4f'\left( x \right)-f'''\left( x \right)<0,\forall x\in \left( 0;1 \right)\) và \(4f\left( x \right)-f''\left( x \right)+m={{x}^{4}}-5{{x}^{2}}+m-11\)

    Nên \(g'\left( x \right)\ge 0,\forall x\in \left( 0;1 \right)\)                

    \(\Leftrightarrow f'\left[ 4f\left( x \right)-f''\left( x \right)+m \right]\le 0,\forall x\in \left( 0;1 \right)\Leftrightarrow f'\left( {{x}^{4}}-5{{x}^{2}}+m-11 \right)\le 0,\forall x\in \left( 0;1 \right)\)

    \(\begin{array}{l} \Leftrightarrow \left[ \begin{array}{l} {x^4} - 5{x^2} + m - 11 \le - 1,\forall x \in \left( {0;1} \right)\\ 0 \le {x^4} - 5{x^2} + m - 11 \le 1,\forall x \in \left( {0;1} \right) \end{array} \right.\\ \Leftrightarrow \left[ \begin{array}{l} m - 10 \le - {x^4} + 5{x^2},\forall x \in \left( {0;1} \right)\\ \left\{ \begin{array}{l} m - 11 \ge - {x^4} + 5{x^2},\forall x \in \left( {0;1} \right)\\ m - 12 \le - {x^4} + 5{x^2},\forall x \in \left( {0;1} \right) \end{array} \right. \end{array} \right.\quad \left( * \right) \end{array}\)

    Xét hàm số \(h\left( x \right)=-{{x}^{4}}+5{{x}^{2}}\) trên \(\left[ 0;1 \right]\)

    Tìm được \(\underset{\left[ 0;1 \right]}{\mathop{\min }}\,h\left( x \right)=0,\)\(\underset{\left[ 0;1 \right]}{\mathop{\max }}\,h\left( x \right)=4.\)

    Do đó 

    \(\begin{array}{l} \left( * \right) \Leftrightarrow \left[ \begin{array}{l} m - 10 \le 0\\ \left\{ \begin{array}{l} m - 11 \ge 4\\ m - 12 \le 0 \end{array} \right. \end{array} \right.\\ \Leftrightarrow \left[ \begin{array}{l} m \le 10\\ \left\{ \begin{array}{l} m \ge 15\\ m \le 12 \end{array} \right. \end{array} \right. \Leftrightarrow m \le 10. \end{array}\)

    \(m\) nguyên thuộc khoảng \(\left( -20;20 \right)\Rightarrow m\in \left\{ -19,...,10 \right\}\)

    \(\Rightarrow \) có 30 giá trị nguyên của m.

    Chọn A

    ADSENSE

Mã câu hỏi: 466296

Loại bài: Bài tập

Chủ đề :

Môn học: Toán Học

Câu hỏi này thuộc đề thi trắc nghiệm dưới đây, bấm vào Bắt đầu thi để làm toàn bài

 
YOMEDIA

Hướng dẫn Trắc nghiệm Online và Tích lũy điểm thưởng

 

 

CÂU HỎI KHÁC

AANETWORK
 

 

YOMEDIA
AANETWORK
OFF